\lecture{02}{2023-10-19}{Topology on $\R$} \gist{% \begin{definition} A set $O \subseteq \R$ is called \vocab{open} in $\R$ iff it is the union of a set of open intervals. A set $A \subseteq \R$ is called \vocab{closed} in $\R$ iff it is the complement of an open set. \end{definition} \begin{remark} \begin{itemize} \item If $\emptyset \neq O \overset{\text{open}}{\subseteq} \R$ then $O \sim \R$. \item If $O \subseteq \R$ is open, then $O$ is the union of open intervals with rational endpoints, since $\Q$ is dense. \end{itemize} \end{remark} \begin{remark}+ $\{O \subseteq \R\} \sim 2^{\aleph_0} < \cP(\R)$. \end{remark} }{} \begin{definition} We call $x \in \R$ an \vocab{accumulation point} of $A$ iff for all $a < x < b$ there is some $y \in A$, $y \in (a,b)$, $y \neq x$. We write \vocab{$A'$} for the set of all accumulation points of $A$. \end{definition} \gist{% \begin{example} $\{\frac{1}{n+1} | n \in \N\}' = \{0\}$. \end{example} }{} \begin{lemma} \label{lem:closedaccumulation} A set $A \subseteq \R$ is closed iff $A' \subseteq A$. \end{lemma} \begin{yarefproof}{lem:closedaccumulation} ``$\implies$'' \gist{% Let $A$ be closed. Suppose that $x \in A' \setminus A$. Then there exists $(a,b) \ni x$ disjoint from $A$. Hence $x \not\in A' \lightning$ }{trivial.} ``$\impliedby$'' Suppose $A' \subseteq A$. \begin{claim} $A \subseteq \R$ is closed iff all Cauchy sequences in $A$ converge in $A$. \end{claim} \gist{% \begin{subproof} Let $A$ be closed and $\langle x_n : n \in \omega \rangle$ a Cauchy sequence in $A$. Suppose that $x = \lim_{n \to \infty} x_n \not\in A$. Then there is $(a,b) \ni x$ disjoint from $A$. However $x_n \in (a,b)$ for almost all $n \in \omega$ $\lightning$ On the other hand let $A$ not be closed. Then there exists a witness $x \in \R \setminus A$ such that $A \cap (a,b) \neq \emptyset$ for all $(a,b) \ni x$. In particular, we may pick $x_n \in (x - \frac{1}{n+1}, x + \frac{1}{n+1}) \cap A$ for all $n < \omega$. \end{subproof} }{} Now if $A' \subseteq A$ and $A$ were not closed, there would be some Cauchy-sequence $(x_n)$ in $A$ such that $\lim_{n \to \infty} x_n \not\in A$. But then $x \in A' \subseteq A \lightning$. \end{yarefproof} \begin{definition} $P \subseteq \R$ (or, more generally, a subset of any topological space) is called \vocab{perfect} iff $P \neq \emptyset$ and $P = P'$. \end{definition} \begin{example}+ Note that being perfect depends on the surrounding topological space: For example, $[0,1] \cap \Q$ is perfect as a subset of $\Q$, but not perfect as a subset of $\R$. \end{example} We want to prove two things: \begin{itemize} \item If $P$ is perfect, then $P \sim \R$. \item If $A$ is closed and uncountable then $A$ has a perfect subset. In particular $A \sim \R$. \end{itemize} \begin{lemma} Let $P \subseteq \R$ be perfect. Then $P \sim \R$. \end{lemma} \begin{proof} \gist{% It suffices to find an injection $f\colon \R \hookrightarrow P$. We have $\underbrace{\{0,1\}^{\omega}}_{\text{infinite 0-1-sequences}} \sim \R$, hence it suffices to construct $f\colon \{0,1\}^\omega\hookrightarrow P$. In order to do that, we are going to construct some $g\colon \underbrace{\{0,1\}^{<\omega}}_{\text{finite 0-1-sequences}} \to P$ with certain properties by recursion on the length of $s \in \{0,1\}^{<\omega}$. Let $g(\emptyset)$ be any point in $P$. Suppose that $g(s) \in P$ has been chosen for all $s$ of length $\le n$. For each $s \in \{0,1\}^{n}$ pick $g(s) \in (a_s, b_s)$ such that $(a_s, b_s) \cap (a_{s'}, b_{s'}) = \emptyset$ for all $s, s'$ of length $n$, $b_s - a_s \le \frac{1}{n^3}$ and $(a_{s\defon{n-1}}, b_{s\defon{n-1}}) \subseteq (a_s, b_s)$. For each such $s$ pick $x_s \in (a_s, b_s) \cap P$ with $x_s \neq f(s)$. This is possible since $P \subseteq P'$. Now set $g(s\concat 0) \coloneqq g(s)$ and $g(s \concat 1) \coloneqq g(x_s)$. This finishes the construction. If $t \in \{0,1\}^{\omega}$, then $(g(t\defon{n}), n < \omega)$ is a Cauchy sequence. By $P' \subseteq P$ we get that this sequence converges to a point in $P$. Define $f(t)$ to be this point. If $t \neq t' \in \{0,1\}^{\omega}$, then there is some $n$ such that $t\defon{n} \neq t'\defon{n}$, hence $f(t) \in [a_{t\defon{n}}, b_{t\defon{n}}]$ and $f(t') \in [a_{t'\defon{n}}, b_{t'\defon{n}}]$ which are disjoint. Thus $f(t) \neq f(t')$, i.e.~$f$ is injective. }{Cantor scheme.} \end{proof}